What is the limit of Gamma function as n approaches infinity?

  • Thread starter Thread starter kq6up
  • Start date Start date
  • Tags Tags
    Approximation
Click For Summary
The limit of the expression Γ(n + 3/2) / (√n Γ(n + 1)) as n approaches infinity is discussed, with Mathematica and Wolfram Alpha providing the limit as 1. A user initially calculated the limit as 1/√e, but later acknowledged an error in their approach. The asymptotic behavior of the Gamma function is clarified, emphasizing that the correct interpretation leads to the limit being 1. The discussion highlights the importance of proper approximations in deriving limits involving the Gamma function.
kq6up
Messages
366
Reaction score
13

Homework Statement



Find the limit of: ##\frac { \Gamma (n+\frac { 3 }{ 2 } ) }{ \sqrt { n } \Gamma (n+1) } ## as ##n\rightarrow \infty ##.

Homework Equations



##\Gamma (p+1)=p^{ p }e^{ -p }\sqrt { 2\pi p } ##

The Attempt at a Solution



Mathematica and wolfram Alpha gave the limit as 1. My solution was ##\frac{1}{\sqrt{e}}##. My work is here https://plus.google.com/u/0/1096789...6080238892798007794&oid=109678926107781868876 Sorry about the photo, but my school is about to set the alarm, and I need to get out of here. Hopefully it is visible.

Thanks,
Chris
 
Physics news on Phys.org
kq6up said:

Homework Statement



Find the limit of: ##\frac { \Gamma (n+\frac { 3 }{ 2 } ) }{ \sqrt { n } \Gamma (n+1) } ## as ##n\rightarrow \infty ##.

Homework Equations



##\Gamma (p+1)=p^{ p }e^{ -p }\sqrt { 2\pi p } ##

The Attempt at a Solution



Mathematica and wolfram Alpha gave the limit as 1. My solution was ##\frac{1}{\sqrt{e}}##. My work is here https://plus.google.com/u/0/1096789...6080238892798007794&oid=109678926107781868876 Sorry about the photo, but my school is about to set the alarm, and I need to get out of here. Hopefully it is visible.

Thanks,
Chris

kq6up said:

Homework Statement



Find the limit of: ##\frac { \Gamma (n+\frac { 3 }{ 2 } ) }{ \sqrt { n } \Gamma (n+1) } ## as ##n\rightarrow \infty ##.

Homework Equations



##\Gamma (p+1)=p^{ p }e^{ -p }\sqrt { 2\pi p } ##

The Attempt at a Solution



Mathematica and wolfram Alpha gave the limit as 1. My solution was ##\frac{1}{\sqrt{e}}##. My work is here https://plus.google.com/u/0/1096789...6080238892798007794&oid=109678926107781868876 Sorry about the photo, but my school is about to set the alarm, and I need to get out of here. Hopefully it is visible.

Thanks,
Chris

You wrote
\Gamma (p+1)=p^{ p }e^{ -p }\sqrt { 2\pi p }\; \Longleftarrow \; \text{FALSE!}
Perhaps you mean ##\Gamma(p+1) \sim p^p e^{-p} \sqrt{2 \pi p}, ##, where ##\sim## means "is asymptotic to". That is a very different type of statement.

Anyway, I (or, rather, Maple) get a different final answer (=1), using the asymptotic result above. Somewhere you must have made an algebraic error.
 
Last edited:
Sorry, yes I meant asymptotic to. Were you able to see my solution via the google plus link? Mary Boas' manual also gives a limit of one. Our solutions start out the same, but I make some approximations that seem not to be justified in her steps. However, I am unable to follow her steps. I can post that too if you are interested in taking a look at her solution.

Thanks,
Chris
 
kq6up said:
Sorry, yes I meant asymptotic to. Were you able to see my solution via the google plus link? Mary Boas' manual also gives a limit of one. Our solutions start out the same, but I make some approximations that seem not to be justified in her steps. However, I am unable to follow her steps. I can post that too if you are interested in taking a look at her solution.

Thanks,
Chris

I don't have her book, and cannot really follow your screenshot. Anyway, if ##r(n)## is your ratio, and using ##\Gamma(p+1) \sim c p^{p+1/2} e^{-p}##, the numerator ##N(n)## has asymptotic form
N(n) = \Gamma(n+3/2) \sim c (n+1/2)^{n+1/2 + 1/2} e^{-(n+1/2)} = c (n+1/2)^{n+1} e^{-n} e^{-1/2}
The denominator ##D(n)## has the asymptotic form
D(n) = \sqrt{n} \: \Gamma(n+1) \sim n^{1/2} c n^{n+1/2} e^{-n} = c n^{n+1} e^{-n}
Therefore,
r(n) \sim \frac{(n+1/2)^{n+1} e^{-n} e^{-1/2}}{n^{n+1} e^{-n}} = \left( 1 + \frac{1}{2} \frac{1}{n} \right)^{n+1} e^{-1/2}
Since ##\lim \,(1 + a/n)^{n+1} = \lim \,(1+a/n)^n = e^a##, we are done: ## \lim r(n) = 1##.
 
  • Like
Likes kq6up
That is pretty much her solution. However, I can actually follow yours better. I made an approximation that was not justified.

Thanks,
Chris
 
Question: A clock's minute hand has length 4 and its hour hand has length 3. What is the distance between the tips at the moment when it is increasing most rapidly?(Putnam Exam Question) Answer: Making assumption that both the hands moves at constant angular velocities, the answer is ## \sqrt{7} .## But don't you think this assumption is somewhat doubtful and wrong?

Similar threads

  • · Replies 7 ·
Replies
7
Views
2K
  • · Replies 24 ·
Replies
24
Views
2K
  • · Replies 14 ·
Replies
14
Views
2K
  • · Replies 8 ·
Replies
8
Views
14K
  • · Replies 3 ·
Replies
3
Views
2K
  • · Replies 8 ·
Replies
8
Views
2K
Replies
13
Views
1K
  • · Replies 5 ·
Replies
5
Views
1K
  • · Replies 12 ·
Replies
12
Views
8K
  • · Replies 6 ·
Replies
6
Views
2K